Suppose Event A is taking 15 or more minutes to get to work tomorrow and Event B is taking less than 15 minutes to get to work tomorrow. Events A and B are said to be complementary events.

a. True
b. False

Answers

Answer 1

Answer:

Hence the answer is TRUE.

Step-by-step explanation:

If event A is taking 15 or more minutes to urge to figure tomorrow and event B is taking but a quarter-hour to urge to figure tomorrow, then events A and B must be complimentary events. this is often because the occurring of 1 is going to be precisely the opposite of the occurring of the opposite event and that they cannot occur simultaneously. In other words, events A and B are mutually exclusive and exhaustive.  

Mathematically,  

P(A) + P(B) = 1.


Related Questions

Based on this example, make a
generalization about the acute angles
formed when two parallel lines are
cut by a transversal.

Answers

Answer:

Step-by-step explanation:

There are 4 of them (acute angles that is)Those 4 are less than 90 degrees.They have supplementary angles which are greater than 90 degrees.There are 4 of them also.The total number of angles should be 8 if there are 2 parallel lines and 1 transversal.

Please helppppppppp!!!!

Answers

Terminal point for 4π/3

(cos4π/3 ,sin4π/3)

{cos(π+π/3) ,sin(π+π/3)}= (-cosπ/3 ,-sinπ/3)

or ,(- 1/2, -√3/2)

OPTION C

Last year Diana sold 800 necklaces. This year she sold 1080 necklaces. what is the percentage increase of necklaces she sold?

Answers

Answer:

13.5% is the increase in percentage

Answer:

74%

Step-by-step explanation:

To get the answer, divide 800 by 1080, and you will get a decimal. That decimal is 0.74074074074. Then, move the decimal point two times two the right, so you should have 074.074074074. Ignore everything after the decimal point as well as the 0 before the decimal point, and if done correctly, it should be 74%.

So, the final answer would be 74%.

Hope this helped!

The product of two numbers is 50 and there sum is 15. Find the number.

Answers

Answer: the numbers are 10 and 5

Step-by-step explanation:

10 times 5 is 50

10 plus 5 is 15

The two numbers are 10 and 5 because 10 times 5 is 50.and 10 plus 5 is 15

Please help asap please

Answers

Answer:

12.9 miles

Step-by-step explanation:

Formula: (x/360)×dπ(circumference)

90/360=1/4

1/4×16.4π

1/4×51.496

12.874

Answer:

[tex]m JM=90 =\Theta[/tex]

[tex]Radius=dimeter/2=16.4/2[/tex]

[tex]\longrightarrowr=8.2[/tex]

The length of arc JM=

[tex]=\frac{\Theta }{360} \times\pi r[/tex]

[tex]=\frac{90}{360} \times2\times\ 3.14\times 8.2[/tex]

[tex]=12.874[/tex]

[tex]\approx 12.9 \; miles[/tex]

[tex]OAmalOHopeO[/tex]

construct an angle that bisect 90°​

Answers

Answer:

We can construct a 90º angle either by bisecting a straight angle or using the following steps.

Step 1: Draw the arm PA.

Step 2: Place the point of the compass at P and draw an arc that cuts the arm at Q.

Step 3: Place the point of the compass at Q and draw an arc of radius PQ that cuts the arc drawn in Step 2 at R.

Step-by-step explanation:

Describe the system of equations
How many solutions does this system have.

Answers

Answer:

Step-by-step explanation:

One solution, at the point of intersection, (3,3)

Simplify
x * x^5 / x^2 * x

Answers

it’s x^3, i guess hope this helps

Hari earns Rs 4300 per month. He spends 80% from his income. How much amount does he save in a year? ​

Answers

Answer:

Hari saves $ 10,320 in a year.

Step-by-step explanation:

Given that Hari earns $ 4300 per month, and he spends 80% from his income, to determine how much amount does he save in a year, the following calculation must be performed:

100 - 80 = 20

4300 x 0.20 x 12 = X

860 x 12 = X

10320 = X

Therefore, Hari saves $ 10,320 in a year.

Find the sum of the geometric series given a1=−2, r=2, and n=8.
A. -510
B. -489
C. -478
D. 2

Answers

Answer:

A. -510

Step-by-step explanation:

We are given the variable values:

a = -2r = 2n = 8

Geometric series formula:

[tex]s = \frac{a( {r}^{n} \times - 1) }{r - 1} [/tex]

Plugging in values we have:

[tex]s = \frac{ - 2( {2}^{8} - 1) }{2 - 1} [/tex]

Simplifying the equation we are left with:

[tex] \frac{ - 2(255)}{1} = - 510[/tex]

someone please help

Answers

Answer:

28

Step-by-step explanation:

78

The answer is 28. Yw !

If 4 gallons of gasoline cost $13.76, how much will 11 gallons of gasoline cost?

Answers

Answer:

x=37.84

Step-by-step explanation:

We can write a ratio to solve

4 gallons        11 gallons

--------------- = ----------------

13.76              x dollars

Using cross products

4x = 11*13.76

4x=151.36

Divide by 4

4x/4 = 151.36/4

x=37.84

Write the point-slope form of an equation of the line through the points (-2, 6) and (3,-2).

Answers

Answer:

[tex]y-6=-\frac{\displaystyle 8}{\displaystyle 5}(x+2)[/tex]

OR

[tex]y+2=-\frac{\displaystyle 8}{\displaystyle 5}(x-3)[/tex]

Step-by-step explanation:

Hi there!

Point-slope form: [tex]y-y_1=m(x-x_1)[/tex] where [tex](x_1,y_1)[/tex] is a point and [tex]m[/tex] is the slope

1) Determine the slope

[tex]m=\frac{\displaystyle y_2-y_1}{\displaystyle x_2-x_2}[/tex] where two given points are [tex](x_1,y_1)[/tex] and [tex](x_2,y_2)[/tex]

Plug in the given points (-2, 6) and (3,-2):

[tex]m=\frac{\displaystyle -2-6}{\displaystyle 3-(-2)}\\\\m=\frac{\displaystyle -8}{\displaystyle 3+2}\\\\m=-\frac{\displaystyle 8}{\displaystyle 5}[/tex]

Therefore, the slope of the line is [tex]-\frac{\displaystyle 8}{\displaystyle 5}[/tex]. Plug this into [tex]y-y_1=m(x-x_1)[/tex]:

[tex]y-y_1=-\frac{\displaystyle 8}{\displaystyle 5}(x-x_1)[/tex]

2) Plug in a point [tex](x_1,y_1)[/tex]

[tex]y-y_1=-\frac{\displaystyle 8}{\displaystyle 5}(x-x_1)[/tex]

We're given two points, (-2, 6) and (3,-2), so there are two ways we can write this equation:

[tex]y-6=-\frac{\displaystyle 8}{\displaystyle 5}(x-(-2))\\\\y-6=-\frac{\displaystyle 8}{\displaystyle 5}(x+2)[/tex]

OR

[tex]y-(-2)=-\frac{\displaystyle 8}{\displaystyle 5}(x-3)\\y+2=-\frac{\displaystyle 8}{\displaystyle 5}(x-3)[/tex]

I hope this helps!

If (-3)^-5 = 1/x, what is the value of x?

Answers

Answer:

-243

Step-by-step explanation:

(-3) (-3) (-3) (-3) (-3) = - 243

[tex]\frac{1}{-243 }[/tex]

The function f(x)=log4x is dilated to become g(x)=f(13x).
What is the effect on f(x)?

Answers

Answer:

f(x) is compressed horizontally

Step-by-step explanation:

Given

[tex]f(x) = \log(4x)[/tex]

[tex]g(x) = f(13x)[/tex]

Required

The effect on f(x)

[tex]g(x) = f(13x)[/tex] implies that f(x) is horizontally compressed by 13.

So, we have:

[tex]f(13) = \log(4 * 13x)[/tex]

[tex]f(13) = \log(52x)[/tex]

So:

[tex]g(13) = \log(52x)[/tex]

You wish to create a 5 digit number from all digits; 0 1 2 3 4 5 6 7 8 9
Repetition is not allowed
* 0 cannot be first as it does not count as a place value if it is first. Ie. 027 is a 2 digit number

How many even numbers can you have?

Answers

Answer:

10234

Step-by-step explanation:

one is the smallest number so its first

and then you can place zero

after that just place the second smallest number

and so on

How many counting numbers have three distinct nonzero digits such that the sum of the three digits is 7?​

Answers

Answer:

6

Step-by-step explanation:

You have 2 conditions.

1. The digits must be different.

2. The digits must add to 7.

There aren't very many

124

142

214

241

412

421

That's it. That's your answer. There are 6 of them

The triangles are similar.


What is the value of x?


Enter your answer in the box.

x =

Answers

Answer:

x=12

Step-by-step explanation:

each side of the smaller triangle, we can multiply by 4 to get the side of the larger triangle

ex: 8*4=32 and 17*4=68

so we can assume that 15*4= 4x+12

60=4x+12

48=4x

x=12

Answer:

x = 12

Step-by-step explanation:

The triangles are similar so we can use ratios

4x+12      32

-------  = ------------

15             8

Using cross products

(4x+12) *8 = 15 * 32

(4x+12) *8 = 480

Divide each side by 8

(4x+12) *8/8 = 480/8

4x+12 = 60

Subtract 12 from each side

4x+12 -12 = 60-12

4x = 48

Divide by 4

4x/4 = 48/4

x = 12

A market surveyor wishes to know how many energy drinks teenagers drink each week. They want to construct a 98% confidence interval for the mean and are assuming that the population standard deviation for the number of energy drinks consumed each week is 1.1. The study found that for a sample of 1027 teenagers the mean number of energy drinks consumed per week is 5.9. Construct the desired confidence interval. Round your answers to one decimal place.

Answers

Answer:

The 98% confidence interval for the mean number of energy drinks consumed per week by teenagers is (5.8, 6).

Step-by-step explanation:

We have that to find our [tex]\alpha[/tex] level, that is the subtraction of 1 by the confidence interval divided by 2. So:

[tex]\alpha = \frac{1 - 0.98}{2} = 0.01[/tex]

Now, we have to find z in the Z-table as such z has a p-value of [tex]1 - \alpha[/tex].

That is z with a pvalue of [tex]1 - 0.01 = 0.99[/tex], so Z = 2.327.

Now, find the margin of error M as such

[tex]M = z\frac{\sigma}{\sqrt{n}}[/tex]

In which [tex]\sigma[/tex] is the standard deviation of the population and n is the size of the sample.

[tex]M = 2.327\frac{1.1}{\sqrt{1027}} = 0.1[/tex]

The lower end of the interval is the sample mean subtracted by M. So it is 5.9 - 0.1 = 5.8 drinks per week.

The upper end of the interval is the sample mean added to M. So it is 5.9 + 0.1 = 6 drinks per week.

The 98% confidence interval for the mean number of energy drinks consumed per week by teenagers is (5.8, 6).

Solve for x and y…….

Answers

The shapes are the same size. Match the sides.

3x -1 = 17

Add 1 to both sides:

3x = 18

Divide both sides by 3:

X = 6

2y = 16

Divide both sides by 2

Y = 8

Answer: x = 6, y = 8

a tree 15m high casts a shadow 8m long. To the nearest degree what is the angle of elevation of the sun?

Answers

Answer:

Answered March 20, 2021

This is a right angle triangle where the hypotenuse a^2 = b^2 + c^2

= 15^2 + 8^2 = 225+64= 289

289= 17^2

17 = hypotenuse

The sine of an angle is the ratio of the shortest side to the hypotenuse

= 8/17= 0.4705

sine^-1 0.4705 = 28°

Which one is greater 4.5% or 0.045

Answers

Answer:

They are equal

Step-by-step explanation:

4.5% is 0.045 in decimal form

Answer: They are equal

Step-by-step explanation:

I always remember by taking the two o's in percent and moving them two spots to the left and vise versa if you want to make a decimal into a percent  (move it two spots to the right).

Function below, choose the correct description of its graph.
vertical
line
horizontal
line
line with a
negative
slope
line with a parabola
positive opening
slope down
O
O
O
O
O
h(x)=0
k(x) = 4x2 +312
f(x) = x-1
O
o
o
O
O
O

Answers

Step-by-step explanation:

I think something went wrong with the answer options you provided. and maybe with the problem statement itself.

I see 3 function definitions.

I can tell you what they are and use the provided option phrasing as closely as possible :

h(x) = 0 is a horizontal line (in fact the x-axis)

k(x) = 4x² + 312 is a parabola with the opening upwards

f(x) = x - 1 is a line with positive slope (going from left to right the line goes up)

Which graph represents the function f (x) = StartFraction 1 Over x + 3 EndFraction minus 2?

Answers

Given:

The function is:

[tex]f(x)=\dfrac{1}{x+3}-2[/tex]

To find:

The graph of the given function.

Solution:

We have,

[tex]f(x)=\dfrac{1}{x+3}-2[/tex]

It can be written as:

[tex]f(x)=\dfrac{1-2(x+3)}{x+3}[/tex]

[tex]f(x)=\dfrac{1-2x-6}{x+3}[/tex]

[tex]f(x)=\dfrac{-2x-5}{x+3}[/tex]

Putting [tex]x=0[/tex] to find the y-intercept.

[tex]f(0)=\dfrac{-2(0)-5}{(0)+3}[/tex]

[tex]f(0)=\dfrac{-5}{3}[/tex]

So, the y-intercept is [tex]\dfrac{-5}{3}[/tex].

Putting [tex]f(x)=0[/tex] to find the x-intercept.

[tex]0=\dfrac{-2x-5}{x+3}[/tex]

[tex]0=-2x-5[/tex]

[tex]2x=-5[/tex]

[tex]x=\dfrac{-5}{2}[/tex]

[tex]x=-2.5[/tex]

So, the x-intercept is [tex]-2.5[/tex].

For vertical asymptote, equate the denominator and 0.

[tex]x+3=0[/tex]

[tex]x=-3[/tex]

So, the vertical asymptote is [tex]x=-3[/tex].

The degrees of numerator and denominator are equal, so the horizontal asymptote is the ratio of leading coefficients.

[tex]y=\dfrac{-2}{1}[/tex]

[tex]y=-2[/tex]

So, the horizontal asymptote is [tex]y=-2[/tex].

End behavior of the given function:

[tex]f(x)\to -2[/tex] as [tex]x\to -\infty[/tex]

[tex]f(x)\to -\infty[/tex] as [tex]x\to -3^-[/tex]

[tex]f(x)\to \infty[/tex] as [tex]x\to -3^+[/tex]

[tex]f(x)\to -2[/tex] as [tex]x\to \infty[/tex]

Using all these key features, draw the graph of given function as shown below.

Answer:

The Answer Is A.

Step-by-step explanation:

Last year, the CDC claimed there were 1700 different strains of a virus around the
world. Since then, numbers have increased by 9.7% more than what the scientists
originally estimated. How many strains are estimated currently? Round to the nearest
number.

Answers

Answer: 1865

Step-by-step explanation:

Given

Claimed strains of virus is 1700

If it is increased by 9.7%

Estimated value can be given by

[tex]\Rightarrow 1700+1700\times 9.7\%\\\Rightarrow 1700(1+0.097)\\\Rightarrow 1700\times 1.097\\\Rightarrow 1864.9\approx 1865[/tex]

Thus, the estimated number is [tex]1865[/tex]

Is the answer right?

Answers

Answer:

one solution.. your answer is correct

Step-by-step explanation:

discriminate = 900 - (4*9*25)  = 0

thus only one solution

your answer is correct

A manager records the repair cost for 14 randomly selected dryers. A sample mean of $88.34 and standard deviation of $19.22 are subsequently computed. Determine the 90% confidence interval for the mean repair cost for the dryers. Assume the population is approximately normal. Step 1 of 2 : Find the critical value that should be used in constructing the confidence interval. Round your answer to three decimal places.

Answers

Answer:

Hence the 90% confidence interval estimate of the population mean is [tex](79.24 , 97.44)[/tex]

Step-by-step explanation:

Given that,  

Point estimate = sample mean = [tex]\bar x[/tex] = 88.34  

sample standard deviation = s = 19.22  

sample size = n = 14  

Degrees of freedom = df = n - 1 = 13  

Critical value =[tex]t\alpha /2,[/tex] df = 1.771

 

Margin of error

[tex]E = t\alpha/2,df \times (\frac{s}{\sqrt{n} } )\\= 1.771 \times (19.22 / \sqrt 14)[/tex]  

Margin of error = E = 9.10  

The 90% confidence interval estimate of the population mean is,  

[tex]\bar x - E < \mu < \bar x + E\\\\88.34 - 9.10 < \mu < 88.34 + 9.10\\\\79.24 < \mu < 97.44\\(79.24 , 97.44)[/tex]

Evaluate the given expression for x = 5 and y=5. 6x2 + 7xy + 3y?​

Answers

Step-by-step explanation:

Given, x = 5

y = 5

= 6(5)^2 +7(5)(5) +3(5)

= 6(25)+7(25) +15

= 150+175 + 15

= 150 + 190

=340

Answer:

x = 12 y = 7

Step-by-step explanation:

6x^2 + 7xy + 3y

6(5)^2+ 7(5) + 7(8)y

6(5+5)+25+35 + 7(8)-7y

60+25+35+ 56-7y

y - 5 = 120 + 35 - 5 (+49y)

sqrt 150 + sqrt 49

x = 12 y = 7

REVISED 2/3/
the following using the picture below.
4
a) Two pairs of supplementary angles:
b) A pair of complementary angles:







Please explain this! Thank you!

Answers

Supplementary angles are those angles which make a sum of 180°.

Complementary angles are those angles which make a sum of 90°.

The supplementary angles are given by the straight lines making angles of 180°.

There are two straight lines CB and DE

The angles DAF and FAE are the two angles making a straight line DE

The angles CAF and FAB are the two angles making a straight line CB

The complementary angles are given by angles formed between the perpendicular lines making angles of 90°.

Angle BAF is formed by angle BAE and angle AEF

Supplementary Angle given by the straight line DE is formed by the angles DAF and FAE.

Complementary Angle BAF is formed by angle BAE and angle AEF.

https://brainly.com/question/12919120

A triangular patch of grass in a park is bordered by walking paths. The longest path bordering the patch of grass measures 110 feet. The smallest path bordering the patch of grass measures 55 feet. The smallest angle formed by the paths bordering the patch of grass measures 29º.
What is the measure of the largest angle of the triangular patch of grass? Round your answer to the nearest
degree. Show all your work.

Answers

Answer:

76 degrees

Step-by-step explanation:

First, we can draw a picture. Two of the sides are 110 feet and 55 feet. In a triangle, the smallest angle is opposite the smallest side and vice versa. Therefore, if I have my triangle arranged in the way shown, the smallest angle of 29 degrees will be opposite of the smallest side of 55 feet.

The law of sines states that a/sinA=b/sinB=c/sinC , with corresponding angles being opposite of its corresponding side. Therefore, we can say that

55 feet/ sin(29 degrees) = 110 / sin(largest angle).

If we say that the largest angle is equal to x, we can say

55 / sin(29°) = 110/sin(x)

multiply both sides by x to remove a denominator

55 * sin(x)/ sin(29°) = 110

multiply both sides by sin(29°) to remove the other denominator

55 * sin(x) = 110 * sin(29°)

divide both sides by 55 to isolate the sin(x)

sin(x) = 110 * sin(29°) / 55

For an angle, if sin(x) = y, we can say that arcsin(y) = x. Therefore, we can say

x = arcsin(110 * sin(29°)/55)

x ≈ 76 degrees

Other Questions
A Gallup survey of 2322 adults (at least 18 years old) in the U.S. found that 408 of them have donated blood in the past two years. Construct a 90% confidence interval for the population proportion of adults in the U.S. who have donated blood in the past two years. You are installing new carpeting in a family room. The room is rectangular with dimensions 20 1/2 feet 13 1/8 feet. You intend to install baseboards around the entire perimeter of the room except for a 3 1/2-foot opening into the kitchen. How many linear feet of board must you purchase? Julattachments.office.net67A car journey is in two stages.Stage 1 The car travels 110 miles in 2 hours.Stage 2 The car travels 44 miles at the same average speed as Stage 1Work out the time for Stage 2Give your answer in minutes.[3 m Describe a balanced one-day menu. Make sure that it meets the daily requirements for the five food groups. Find the mean of first 5 prime numbers. Pam and Amanda and Mike work at different clothing stores. Amanda made twice what Pam earned and Juliemade $90 more than Amanda. If their total earnings for a week are $995, how much did each person make? what is the volume if the radius is 7m and the height is 9m. a handbag was purchased for Rs.280/- and sold for Rs.350/- calculate profit% Learning Task 4: Write the correct word/words to complete the answers to the questions about greenhouse effect.What are the greenhouse gases? 1._____________, 2.______________, 3.________________What do greenhouse gases do? (4.) ____________ the atmosphere by trapping (5). _________.What is the greenhouse effect? The (6). ____________________ of the atmosphere because of insulation by ____________________________How does the greenhouse effect work? Incoming (7)_____________________ from the sun (8)______________ the Earth and is (9)__________________by the greenhouse gases, warming and insulating the Earth like a blanketIs the greenhouse effect a good thing? (10)_______!! Because it maintains _(11)___________________So why is the greenhouse effect considered a bad thing and is linked to global warming? (12)______________ of the greenhouse effect is a bad thing. Since there are more and more greenhouse gases in the atmosphere, more (13)_______________ is trapped which makes the earth (14)____________. This is called (15)___________________________.gagawin ko pong brainliest ang makakasagot ng tama, pa answer po plsss... ASAP PO TY2SEE ANSWERS please help !!!12. JKL = ONM by SAS. Select one set of corresponding parts that could be marked congruent by CPCTC. What are the values of x and y? A) X= 9/2; y = 9 B) x = 1843 ; y= 983 C) x= 9; y = 983 OD) x= 913 ; y=9 If f of x equals the quotient of the quantity x squared minus 36 and the quantity x plus 6 is continuous at x = -6, find f(-6). Which word from the passage gives the best indication of the speakers tone?dreamsscreamwingssing What are the equations of the asymptotes for the functiony=tan2pix where 0 Julie has 5.00 and wants to buy some glue at Walmart glue cost 4.25 how much will julie get back in change and you have to subtract a d slove problem why election campaign is required for democratic elections On whom the trade bill drawn ? could you tell me howto do pls A 1400-seat theater sells two types of tickets for a concert. Premium seats sell for $30 each and regular seats sell for $20 each. At one event $33,950 was collected in ticket sales with 10 seats left unsold. How many of each type of ticket was sold? Can someone help me out please